Someone please help me.

Someone Please Help Me.

Answers

Answer 1

Answer:

1 is good number 2 is wrong number 3 is good

Step-by-step explanation:

your welcome im juss writting more so it wont say issa short question!!


Related Questions

Given the points A(-5, -6) and B(8, 2), if C is the midpoint of segment AB, find the coordinates of the midpoint of segment CB

Answers

It’s c bucko I did that

The rate of auto thefts doubles every 5 months.
(a) Determine, to two decimal places, the base b for an exponential model y = Abt of the rate of auto thefts as a function of time in
months.
(b) Find the tripling time to the nearest tenth of a month.
months

Answers

I think first one is the answer

mark me as branlist and follow me

Answer the questions below.
2.
(a) The area of a rectangular parking lot is 6370 m
If the width of the parking lot is 65 m, what is its length?

Answers

The length = 98 because 6370 divided by 65 is 98

The value of -31.45/-3.21 is around?

Answers

My calculator says 9.797507788, so you can just round to 9.8

Please help. Don’t understand this math problem

Answers

Answer:

-24r + 56

Step-by-step explanation:

To find the area of this rectangle we multiply width and length

8 × (-3r + 7) = -24r + 56

Which of the following inequalities is incorrect?
00>-2
6 >-12
-5<-8
0-7<-5

Answers

Step-by-step explanation:

First, is 0 greater than -2? Correct

Next, is 6 greater than -12? Correct

Is -7 less than -5? Correct

Finally, is -5 less than -8? NERP!!

So C. is incorrect. Or in this case, it is...

What is the equation shown in standard form of the line shown on the graph?
O x = 4
O y = 4
O x - y = 4​

Answers

Answer:

[tex]y = 4[/tex]

Step-by-step explanation:

Because no matter what value x is, y is always 4

The price of a stock was $325 a share. The price of the stock went down $25 each week for 6 weeks what was the price of that stock at the end of 6 weeks?

Answers

Answer:325 - (25 x 6 ) = $175

Step-by-step explanation:

fill in the table using the function rule: y=1 + 4x​

Answers

Answer:

13, 25, 37, 41

Step-by-step explanation:

By plugging each x value given for x, you are able to solve for y. Just multiply x by 4 and add 1.

13-25-36-41 is the answer

Help solve step by step please? 3/7➗4/5=

Answers

Answer:

15/28

Step-by-step explanation:

Step-by-step explanation:

3/7 divided by 4/5

Find the reciprocal of 4/5 which is 5/4

Multiply 3/7 by 5/4

3*5= 15 which is the numerator

7*4=28 which is the denominator

so the final answer is 15/28

What is the property of (4y + 1) x 0 =0

Answers

Answer: Zero Property

Step-by-step explanation:

Graph the linear equation by finding its intercepts.
3x + 6y = 18

Answers

x intercepts are (6,0)
y intercepts are (0,3)

OMG! I just answered the wrong question... so sorry

A box of books weighs 4.10 kg for every meter of it's height. Convert this ratio into pounds per foot.

Answers

Answer:

The ratio is   k  =   2.76 pounds /  foot

Step-by-step explanation:

From the question we are told that

   For  1 m  height  =>  4.10 kg mass

generally  1 meter = 3.28084 feet  

Also

  1 kg  = 2.20462 pound

  4.10 kg  =  x pounds

So

      [tex]x = \frac{4.10 * 2.20462}{1 }[/tex]

=>    [tex]x = 9.04 \ pounds[/tex]

So

        9.04 pounds  = 3.28084  feet  

         z pounds  =  1 foot

So  

       [tex]z = \frac{ 9.04 * 1 }{ 3.28084}[/tex]

=>   z = 2.76 pounds

So the ratio is in pounds per foot is

      k  =   2.76 pounds /  foot

       

which decimal is less than 0.8 and greater than 0.02

Answers

Answer:

0.02<x<0.80

0.4

Step-by-step explanation:

0.02 is equal to 2/100 and 0.8 is equal to 80/100

so you could really choose any number between 0.03 and 0.79

Answer:

Less than 0.8 and greater than 0.02

0.8 > a > 0,02

There are many decimals:

0.7

0.67

0.6

0.5

0.4

0.3

0.2

0.1

0.023

0.021

and many others

what value of x makes this equation true X over 3 = 24

Answers

Answer:

x=72

Step-by-step explanation:

Start with [tex]\frac{x}{3}[/tex]=24

Using the multiplication property of equality, multiply each side by 3

x=72

The answer is 72 it would be 72

Which statements are true regarding f(x)=10/x+5

? Check all that apply.

The domain of f(x) is (–∞, –5) U (–5, ∞).
The range of f(x) is (–∞, 2) U (2, ∞).
The x-intercept is at (–5, 0).
The y-intercept is (0, 2). 
There is a vertical asymptote at x = –5.
The end behavior is  x → –∞, f(x) → 0 and x → ∞, f(x) → 0.

Answers

For the function   [tex]\frac{10}{x + 5}[/tex]  , we can say

The domain of f(x) is (–∞, –5) U (–5, ∞).

The y-intercept is (0, 2).

There is a vertical asymptote at x = –5.

The end behavior is  x → –∞, f(x) → 0 and x → ∞, f(x) → 0.  

What is a function?

A mathematical relationship from a set of inputs to a set of outputs is called a function.

What is domain of a function?

The domain of a function is the set of all possible inputs for the function.

What is range of a function?

The range of a function is the set of outputs the function achieves when it is applied to its whole set of outputs

What is asymptote?

An asymptote is a straight line that can be horizontal, vertical or oblique that goes closer and closer to a curve which is the graphic of a given function.

What is intercept?An intercept of any function is a point where the graph of the function crosses, or intercepts, the x-axis or y-axis.To find the x-intercept we need to do y = 0To find the y-intercept, we need to do x = 0

How to know the domain of function?

The given function is  [tex]\frac{10}{x + 5}[/tex]  

Now, if x = -5 then the function becomes undefined.

∴ The domain of the function is all real numbers except -5

So, The domain of f(x) is (–∞, –5) U (–5, ∞).

So, option a is correct.

How to know the range of the function?

Clearly, the range of the function includes all real numbers

So, option B is wrong

How to find the x-intercept?

To find the x-intercept , we will have to solve f(x) = 0,  which is not at all possible for this function.

So, option C is wrong

How to find the y-intercept?To find the y-intercept, we need to do x = 0

So, putting x = 0 in the function we get f(x) = 2

So, the y- intercept is (0, 2).

So, option D is correct.

How to find vertical asymptote ?To find the vertical asymptote, we need to make the                    denominator = 0

∴ x+ 5 = 0

∴ x = -5

So, there is a vertical asymptote at x = –5.

Hence option E is correct.

How to find the end behavior?

We can see,

As x tends to -∞, x + 5 will also tend to -∞We know that a number divided by ∞ or -∞ becomes 0

So, as x tends to -∞, f(x) will tend to 0

So, Option F is correct.

Find more about "Domain and Range of Functions" here: https://brainly.com/question/1942755

#SPJ2

Answer:

ADEF

Step-by-step explanation:

ADEF

Find the sum of the squared residuals for each potential line of fit, and then determine which potential line is the true line of best fit. Equation 1 Square of residual .28 .167 .19 Equation 2 Square of residual .005 .110 .09 Equation 3 Square of residual .125 .125 .26 Equation 4 Square of residual .212 .009 .08 Question 8 options: Equation 4 Equation 1 Equation 3 Equation 2

Answers

Answer: Equation 2

Step-by-step explanation:

Equation 1 Square of residual .28 .167 .19

Sum of Square Residual :

( .28 + .167 + .19) = 0.637

Equation 2 Square of residual .005 .110 .09

Sum of Square Residual :

(.005 + .110 + .09) = 0.205

Equation 3 Square of residual .125 .125 .26

Sum of Square Residual :

( .125 + .125 + .26) = 0.51

Equation 4 Square of residual .212 .009 .08

Sum of Square Residual :

( .212 + .009 + .08) = 0.301

Equation 2 is the best potential line of best fit as it has lowest value of sum of square residual. And the best line of fit is one which the sum of Squared error is lowest.

27. Give the perimeter of the rectangle below in
simplest form.

Answers

Answer:

The permeter will be 26 units

Step-by-step explanation:

First to find this you need to know what the value of x is to do this I will use the equation 3x - 7 = x + 2

1. add seven to both sides leaving 3x = x + 9

2. Then subtract the 1x from both sides leaving 2x = 9

3. Then divide 9 by 2 which will be 4.5, x = 4.5

Then just plug 4.5 into each x value 4.5 + 2 and 3(4.5) - 7

1. 4.5 + 2 = 6.5

2. 3(4.5) - 7              13.5 - 7 = 6.5

The perimeter of the rectangle below in simplest form will be 4(x - 3).

Length of the rectangle = 3x - 7

Width of the rectangle = x+2

The perimeter of a rectangle is given as 2(length + width). The perimeter of the rectangle given will then be:

= 2(length + width)

= 2(3x - 7) + 2(x + 2)

= 6x - 14 + 2x + 4

= 4x - 12 = 4(x - 3)

Therefore, the perimeter of the rectangle in simplest form will be 4(x - 3).

Read related link on:

https://brainly.com/question/24429956

The manufacturer of pencils spends $0.05 to make each pencil and sells them for 0.25. The manufacturer also has fixed
costs each month of $2000. Find the number of pencils x that the manufacturer needs to sell in order to break even

Answers

Answer:

x=10000

Step-by-step explanation:

The break even point occurs when costs equal revenue. Write the break-even formula and solve to find

C(x)0.05x−20000.2xx=R(x)=0.25x=2000=10000.

A rectangle is 3 times as long as it is wide. The perimeter is 24 feet. Find the dimensions.

Answers

Answer:

Width is 3 units, The Length is 9 units.

Step-by-step explanation:

Three times as much as 3, or 9, is how you find the lengths. This being- 3+3=6 and 9+9=18, 6+18=24, which is the perimeter, therefore 3 and 9 are the dimensions.

Please Help A) Find the perimeter B) The area of the rectangle

Answers

Answer:

A) 36.2m  B) 70m

Step-by-step explanation:

5.6m + 5.6m = 11.2 m

12.5 + 12.5= 25  11.2 + 25 = 36.2m

Length x width equals area

12.5m = length and 5.6m= width

multiply 12.5 x 5.6= 70m

What is the value of the digit 5 in the number 4.105

Answers

Answer:

thousanths place

Step-by-step explanation:

Answer:

4.105

5-hundredths place if I’m correct

Assuming that a local cost cutter provides cuts perms and hair styling service and you fix cost are $120,000 in the variable cost or 40% of the sales revenue last year’s revenue total of $250,000. What is the break even points in sales dollars?

Answers

Answer:

Unit selling price

$7 $9 $7

Unit variable costs

(4) (5) (1)

Unit contribution margin

$3, $4, $6

With monthly fixed costs of $306,000, the company sells two units of A for each unit of B and three units of B for each unit of C.


Is the following number an integer?
30/6

Answers

Answer:

NO ! For 30, the answer is: No, 30 is not a prime number. The list of all positive divisors (i.e., the list of all integers that divide 30) is as follows: 1, 2, 3, 5, 6, 10, 15, 30.

Step-by-step explanation:

CE is an angle bisector of angle ACB.
If m angle ACE = 2x - 15 and m angle ECB = x +5,
determine the value of x.

Answers

Answer:

x = 20

Step-by-step explanation:

[tex] \because \overrightarrow {CE}[/tex] is an angle bisector of[tex] \angle ACB. [/tex]

[tex]\therefore m \angle ACE = m \angle ECB\\ 2x - 15 = x + 5 \\ 2x - x = 15 + 5 \\ \huge \orange{ x = 20}[/tex]

The value of x from the angle bisector is: x = 20

How to find the value of the angle bisector?

Angle bisector theorem is a theorem in geometry that states that an angle bisector of a triangle divides the opposite side into two segments that are proportional to the other two sides of the triangle. An angle bisector is a ray that divides a given angle into two angles of equal measures.

Thus, we can say that:

∠ECB = ∠ACE

Thus:

x + 5 = 2x - 15

2x - x = 15 + 5

x = 20

Read more about angle bisector at: https://brainly.com/question/24334771

#SPJ2

Help me please thanks

Answers

Answer: B. M= 6

Step-by-step explanation:

4(6-1)-5=2(6)+3

The first part of the equation, 4(6-1)-5, 6-1 is 5 and then you multiply that 5 by 4 and get 20, and subtract 5 from 20 and get 15. The second part, 2(6)+3, 6 x 2 is 12 and then you add 3 which equals 15. 15=15 so the answer is correct

A candy store sells mints in 14, 12 , and 34 lb packages. John purchases several packages of each weight.

What is the weight of 6 packages of the 14 lb package?

Answers

Answer:

84 lbs

Step-by-step explanation:

Multiply 14*6 to get the total weight

Which of the following trinomials could represent the product (x + m)(x + n) if m = 5 and n = -7? A x2 + 2x - 35 B x2 - 12x - 35 Cx2 + 35x + 2 D x2 - 2x - 35

Answers

Hey there! I'm happy to help!

Let's plug in these values.

(x+5)(x-7)

We use the FOIL method to multiply the two.

We multiply the Firsts.

x·x=x²

We multiply the Outsides.

-7·x=-7x

We multiply the Insides.

5·x=5x

And we multiply the Lasts.

-7·5=-35

We combine these all.

x²-7x+5x-35

x²-2x-35

Have a wonderful day! :D

Evaluate =f(×) r 6x-2 for f (2a-3)​

Answers

The first option 12a - 20

Answer:

[tex] \boxed{\sf f(2a - 3) = 12a - 20} [/tex]

Step-by-step explanation:

Function given:

⇒f(x) = 6x - 2

Substitute 2a - 3 into the function in place of x:

⇒f(2a - 3) = 6(2a - 3) - 2

Expand bracket:

⇒f(2a - 3) = 12a - 18 - 2

Add the like terms:

⇒f(2a - 3) = 12a - 20

James has 1,836 marbles. He decides to divide them equally among 23 boxes. How many marbles will James have left over?

Answers

Answer:

79.8260869565

Step-by-step explanation:

When dividing 1,836 by 23= 79.8260869565

Other Questions
differences and similarities to a political and physical map which statement from Barrio boy states the most reasonable inference and provides a valid clue to support it victoria es inteligente y Select the reasons why men joined the crusades:adventurepolitical advancementwealthreligious self-determinationfree Jerusalem from the Turksto learn about Islam PLEASE HELP 28 POINTS PLUS BRAINLYIST!!! MUST ANSWER ALL OR AS MANY AS YOU KNOW. THANKS. Martha has 90 red tiles and 72 yellow tiles. She wants to place the tiles in rows using one color for each row without having any tiles leftover. Each row must have the same number of tiles. What is the greatest number of tiles Martha can use for each row? can someone help me please Which of the following could be the first sentence in a paragraph proof of the statement below?If 2x + 5 = 11, then x = 3(point)A.The solution of the given statement is 2 = 3..BThe solution of the given statement is 23 + 5 = 11.C.The given statement is x = 3.D.The given statement is 2x + 5 = 11.Can someone help me with this asap 4/5 x (-2/9) (as a fraction) (simplified) B. What is (64) ? What is (19) ? Carbohydrates are an important type of _ compound.A) organicB) lipidsD) protein what is 8n to the 3 power times n to the 4th power? Idk which one its hard Mental Math In a toy store, the ratio of the number of dolls to the number of teddy bears is 6:5. If thestore has 240 dolls, how many teddy bears are in the store? Use pencil and paper. Describe themental math steps you use to solve this problem.There are teddy bears in the store. Which of these sentences best summarizes the passage below (paragraph 6)?"And the attraction that many of us felt, the desire to smell one another's smells, feel or rub or caress one another's scales or skin or features or fur, taste one another's blood or flesh, keep one another warm - that attraction was now all one with the fear, and the hunter could not be told from the hunted, not the eater from the food."A. The animals revolted when they realized they no longer had names.B. Unnaming the animals proved to be a mistake since it incited fear among the creatures.C. Unnaming the animals caused confusion since the animals didn't know who to hunt.D. The animals were no longer separated by labels and wanted to interact with each other. How is it useful to identify emerging patterns in looking for a trends? what was the reasonings for the 9/11 attack reflect (9, -3) across the y-axis. Then reflect the resukt across the x-axis. what are the coordinates to the final point SOMEONE PLS HELP WITH THESE TWO QUESTIONS1)Chads family just moved into a new home. The sun went down as Chad was moving boxes into the house, so he had to turn on a light to see. Which statements are true about the power Chad used? Choose the two statements that apply. A. The power he used to move the boxes can be found by determining the rate at which work was done on the boxes. B. The power Chad used to move the boxes is the force he used times the distance he moved them. C. The power of the light Chad turned on is the amount of energy changed from electricity to light. D. The power of the light Chad turned on is the amount of energy transferred per time.2)Raul dug a hole in his yard to repair a water pipe. It took him 2 seconds to apply a force of 50 Newtons to push the shovel 0.25 m into the ground. How much power was used? A. 0.01 watts B. 6.25 J/s C. 25.00 Nms D. 100.00 watts No soy serio, ____ me gusta leer.